Strengthen with Sufficient Premise Questions - - Question 4

Modern physicians often employ laboratory tests, in addition to physical examinations, in order to diagnose diseases ...

lostsoul December 11, 2016

Misunderstanding

I'm not sure if I am understanding this question correctly...

Replies
Create a free account to read and take part in forum discussions.

Already have an account? log in

Mehran December 20, 2016

@lostsoul this is a Strengthen with Sufficient question so we are looking for the answer choice that 100% guarantees the conclusion.

What is the conclusion?

"Insurance company regulations that deny coverage for certain laboratory tests therefore decrease the quality of medical care provided to patients."

The support for this conclusion?

"Modern physicians often employ laboratory tests, in addition to physical examinations, in order to diagnose diseases accurately."

So we are looking for the answer choice that 100% guarantees that insurance company regulations that deny coverage for certain laboratory tests decreases the quality of medical care provided to patients.

The gap we need to close here is the denial of certain laboratory tests and a reduction in the quality of medical care provided to patients.

(A) closes this gap by stating, "Physical examinations and the uncovered laboratory tests together provide a more accurate diagnosis of many diseases than do physical examinations alone."

By stating that physical examinations + uncovered laboratory tests provide a more accurate diagnosis of many diseases than just physical examinations alone, this would serve to justify the conclusion that denial of coverage for certain laboratory tests decreases the quality of medical care provided to patients.

Hope that helps! Please let us know if you have any other questions.

justicelong94 August 24, 2017

After reading this question, I was stuck between A and C and I chose C. I still do not understand how C is not the better answer choice. I agree with conclusion and the basis for the conclusion. So C states that "Many patients who might benefit from the uncovered laboratory tests do not have any form of health insurance." This statement alone strengthens the argument by saying that if had health insurance they would benefit from the uncovered laboratory test. When I negate it, I came up with "Many patients who might benefit from the uncovered laboratory test do have health insurance; which 100% guarantees the conclusion that "insurance company regulations that deny coverage for certain laboratory tests decrease the quality of medical care provided to patients". I am not sure how C is not the answer

Mehran August 26, 2017

Ah--you are trying to negate an answer choice as though this was a Strengthen with Necessary premise question, but it is not--it is a Strengthen with Sufficient question. This means, as explained above, that you are looking for the answer choice that 100% guarantees the conclusion.

The conclusion is: "Insurance company regulations that deny coverage for certain laboratory tests therefore decrease the quality of medical care provided to patients."

The only premise offered in support is "Modern physicians often employ laboratory tests, in addition to physical examinations, in order to diagnose diseases accurately."

What's the gap? The conclusion assumes that the denial of certain laboratory tests means that patients are receiving lower quality medical care.

(C) says "Many patients who *might* benefit from the uncovered laboratory tests do not have any form of health insurance."

Those people are screwed--but in a way that has absolutely no bearing on this particular question or the conclusion at hand. The conclusion here is about insurance company regulations--and so is limited to the role that health insurance companies play in determining quality of medical care available to patients.

Put differently--even if the patients described in (C) had health insurance, their insurance companies would deny coverage for certain lab tests. This does not strengthen the argument in the stimulus.

The real problem here is the assumption identified above. Only answer choice (A) closes that gap.

Hope this helps! Please let us know if you have any additional questions!

mattjwoolslayer April 7, 2019

How do we know the difference between a Strengthen with necessary premise versus strengthen with sufficient in the question stimulus. I get it if there is an "if' when the question is asked but i am having difficulty when there is not.

Ravi April 8, 2019

@mattjwoolslayer,

Happy to help.

The two question types you're referring to are strengthen with a
necessary premise and strengthen with a sufficient premise. Let's take
a look at how they differ.

1) strengthen with a necessary premise questions

These questions typically say things like

Which one of the following is an assumption on which the argument depends?

Which one of the following is an assumption required by the
editorial's argument?

The consumer's argument relies on the assumption that

Which one of the following is an assumption on which the argument depends?

Which one of the following is an assumption required by the
researcher's argument?

The key giveaway in these question stems that tell us they're
strengthen with a necessary premise questions is that they all contain
wording that tells us that we're looking for an assumption that the
argument NEEDS/REQUIRES/HAS TO HAVE in order for it to have any chance
at holding up.

Necessary premises are premises that must be true in order for the
argument to hold. In other words, if a necessary premise is false,
then the argument falls apart.

2) strengthen with a sufficient premise questions

These question types typically say things like

Which one of the following, if assumed, enables the psychologist's
conclusion to be properly drawn?

The ethicist's conclusion follows logically if which one of the
following is assumed?

The argument's conclusion follows logically if which one of the
following is assumed?

Which one of the following, if assumed, enables the essayist's
conclusion to be properly drawn?

The conclusion of the criminologist's argument is properly inferred if
which one of the following is assumed?

The key giveaway in these question stems that tell us they're
strengthen with a sufficient premise questions is that they all
contain language that tells us that we're looking for an assumption
that, if true, ENABLES THE CONCLUSION TO BE DRAWN/JUSTIFIES THE
ARGUMENT, etc. In other words, we're looking for an assumption that,
if true, makes the argument valid.

Sufficient premises are premises that, if we add them to the argument,
automatically make the argument valid.

To recap, think of necessary premises as premises that, if false, make
the argument lose. Think of sufficient premises as premises that, if
true, make the argument win (valid).

Does this help? Let us know if you have any more questions!